UWorld Neuro

¡Supera tus tareas y exámenes ahora con Quizwiz!

The nurse receives report for 4 clients in the emergency department. Which client should be seen first? 1. 30-year-old with a spinal cord injury at L3 sustained in a motor cycle accident who reports lower abdominal pain and difficulty urinating 2. 33-year-old with a seizure disorder admitted with phenytoin toxicity who reports slurred speech and unsteady gait 3. 65-year-old with suspected brain tumor waiting to be admitted for biopsy who reports throbbing headache and had emesis of 250 mL 4. 70-year-old with atrial fibrillation and a closed-head injury waiting for brain imaging who reports a headache and had emesis of 200 mL

4 A client with a neurological injury (eg, head trauma, stroke) is at risk for cerebral edema and increased intracranial pressure (ICP), a life-threatening situation. The client with atrial fibrillation may also be taking anticoagulants (eg, warfarin, rivaroxaban, apixaban, dabigatran), making a life-threatening intracranial bleed even more dangerous. The nurse should perform a neurologic assessment (eg, level of consciousness, pupil response, vital signs) immediately. (Option 1) Autonomic dysreflexia (eg, throbbing headache, flushing, hypertension) is a life-threatening condition caused by sensory stimulation that occurs in clients who have a spinal cord injury at T6 or higher. This is not the priority assessment as this client's injury is at L3. This client likely has acute urinary retention and needs catheterization. (Option 2) Phenytoin toxicity commonly presents with neurologic manifestations such as gait disturbance, slurred speech, and nystagmus. These are expected symptoms and therefore are not a priority. (Option 3) A brain tumor can also cause increased intracranial pressure; clients report morning headache, nausea, and vomiting. Dexamethasone (Decadron) can be prescribed short-term to decrease the surrounding edema. A tumor usually grows more slowly than a possible hematoma and is therefore not the priority assessment. Educational objective: Constant headache, decreased mental status, and sudden-onset emesis indicate increased intracranial pressure.

A speeding driver sustained a closed-head injury in an acceleration/deceleration accident from striking a tree front end first. Based on the coup-contrecoup phenomenon, which assessments are most likely to be affected related to the involved areas of the brain? 1. Expressive speech, vision 2. Light touch, hearing 3. Sense of position, graphesthesia 4. Weber tuning fork test, cranial nerve I

1 Coup-contrecoup injury occurs when a body in motion stops suddenly (eg, head hits car windshield), causing contusions (bruising) of brain tissue as the brain moves back and forth within the skull. First, the soft tissue strikes the hard skull in the same direction as the momentum (coup). As the body bounces back, the brain strikes the opposing side of the skull (contrecoup). When the forward collision occurred, the frontal lobe most likely suffered the primary impact (coup). Executive function, memory, speech (Broca area), and voluntary movement are controlled by the frontal lobe. The contrecoup most likely injured the occipital lobe, where vision is processed. (Option 2) The temporal lobe (lateral aspect of the brain) controls hearing and integrates sensory data (eg, auditory, visual, somatic). The Wernicke speech area in the temporal lobe is responsible for language comprehension. Light touch is processed by the sensory cortex in the parietal lobe. (Option 3) An interruption of sensory function indicates injury to either the spinal column or the parietal lobe. These injuries affect proprioception (awareness of body positioning) and graphesthesia (ability to identify writing on the skin, by touch). (Option 4) The Weber test screens for conductive hearing loss by checking whether a tuning fork held along the midline of the head is heard evenly in both ears. Cranial nerve I is the olfactory nerve. Hearing and smell are both processed by the temporal lobe. Educational objective: Coup-contrecoup injuries usually affect the frontal and occipital lobes. The frontal lobe controls executive function, memory, speech, and motor skills. The occipital lobe processes vision.

A highly intoxicated client was brought to the emergency department after found lying on the sidewalk. On admission, the client is awake with a pulse of 70/min and blood pressure of 160/80 mm Hg. An hour later, the client is lethargic, pulse is 48/min, and blood pressure is 200/80 mm Hg. Which action does the nurse anticipate taking next? 1. Administer atropine for bradycardia 2. Administer nifedipine for hypertension 3. Have CT scan performed to rule out an intracranial bleed 4. Perform hourly neurologic checks with Glasgow coma scale (GCS)

3 Cushing's triad is related to increased intracranial pressure (ICP). Early signs include change in level of consciousness. Later signs include bradycardia, increased systolic blood pressure with a widening pulse pressure (difference between systolic and diastolic), and slowed irregular (Cheyne-Stokes) respirations. Cushing's triad is a later sign that does not appear until the ICP is increased for some time. It indicates brain stem compression. The skull cannot expand after the fontanels close at age 18 months, so anything taking up more space inside the skull (eg, hematoma, tumor, swelling, etc.) is a concern for causing pressure on the brain tissue/brain stem and potential herniation. In this scenario, hidden head trauma causing an intracranial bleed must be ruled out with diagnostic testing. The client's intoxication could blunt an accurate history or presentation of a head injury. (Option 1) Atropine is used to stimulate the sinoatrial node in bradycardia with systemic symptoms. An electrocardiogram (ECG) should be obtained prior to administering atropine. In this client, there is no evidence of a cardiac etiology or systemic symptoms of poor perfusion/circulation from the bradycardia. (Option 2) Nifedipine (Procardia) is a calcium channel blocker that is a potent vasodilator. However, all components must be considered in this scenario as to the etiology of the hypertension rather than just treating that sign. Ruling out a cerebral cause of the hypertension is most important. (Option 4) The nurse would continue to perform neurologic assessments (including GCS). However, it is more important to obtain appropriate diagnostic tests and initiate treatment for the changing neurologic symptoms than to just monitor and document. In addition, the nurse should be performing these assessments more frequently than hourly in this rapidly changing client. Educational objective: Cushing's triad/reflex indicates increased intercerebral pressure. Classic signs include bradycardia, rising systolic blood pressure, widening pulse pressure, and irregular respirations (such as Cheyne-Stokes).

The nurse is caring for a client with left-sided weakness from a stroke. When assisting the client to a chair, what should the nurse do? 1. Bend at the waist 2. Keep the feet close together 3. Pivot on the foot proximal to the chair 4. Use a transfer belt

4 When transferring a client from bed to chair the following are recommended for client safety: - Clients should wear nonskid shoes (first step) - Make sure the bed and chair (wheelchair) brakes are locked - Use a transfer belt. A transfer belt worn around the client's waist allows the nurse to assist the client while maintaining proper body mechanics and safety. - Transfer the client toward the stronger (not the weaker) side. If the client is weak on the left side, ask the client to pivot on the right side. (Option 1) Bending at the waist often requires the nurse to use the back for lifting, making for poor body mechanics. (Option 2) The nurse should provide a wide body stance for more stability. Keeping the feet close together would not be good body mechanics and could cause injury. (Option 3) The nurse using proper body mechanics would pivot on the foot distal to the chair. Educational objective: A transfer belt worn around the client's waist allows the nurse to assist the client while maintaining proper body mechanics and safety.

The client comes to the emergency department status post fall. The client is squinting both eyes and reports sudden blurry vision. The nurse is aware that this deficit reflects injury to which area of the brain?

The occipital lobe of the brain registers visual images. Injury to the occipital lobe could result in a deficit with vision. The nurse should notify the health care provider immediately and document the finding. The frontal lobe controls higher-order processing, such as executive function and personality. Injury to the frontal lobe often results in behavioral changes. The temporal lobe integrates visual and auditory input and past experiences. The parietal lobe integrates somatic and sensory input. Educational objective: The occipital lobe receives visual images. The frontal lobe controls executive function and personality. The temporal lobe receives auditory input. The parietal lobe receives sensory input.

The nurse is planning care for a client being admitted with newly diagnosed quadriplegia (tetraplegia). Which intervention will the nurse prioritize? 1. Assess vital capacity and tidal volume once per shift and PRN 2. Perform passive range of motion exercises on affected joints every 4 hours 3. Provide time during each shift for the client to express feelings 4. Turn the client every 2 hours throughout the day and night

1 Quadriplegia (tetraplegia) occurs when the lower limbs are completely paralyzed and there is complete or partial paralysis of the upper limbs. This is usually due to injury of the cervical spinal cord. Depending on the area of injury and extent of cord edema, the airway can be adversely affected. The priority assessment for this client is the status of the airway and oxygenation. The nurse should frequently assess breath sounds, accessory muscle use, vital capacity, tidal volume, and arterial blood gas values (if prescribed). (Options 2, 3, and 4) This client will need passive range of motion exercises on affected joints to prevent contractures. Turning the client will be necessary to prevent skin breakdown over bony prominences. The client will need to express feelings and work through the grief process related to loss of function. Bladder and bowel training programs will be necessary. All of these interventions are important for this client but are not the priority over maintenance of adequate oxygenation. Educational objective: The priority assessment in a client newly diagnosed with quadriplegia (tetraplegia) is airway management and oxygenation.

The nurse is caring for a client in the medical-surgical unit who has delirium according to the Confusion Assessment Method assessment tool. Which of the following assessment findings are likely contributing to the client's delirium? Select all that apply. 1. Multiple doses of IV hydromorphone administered in the past 12 hours 2. Serum sodium of 123 mEq/L (123 mmol/L) 3. SpO2 of 82% on room air 4. Temperature of 103.1 F (39.5 C) 5. Urine culture positive for gram-positive cocci in chains

ALL Delirium is characterized by an acute or fluctuating change in mental status that is often reversible and related to an underlying medical condition. Evidence-based assessment tools, such as the Confusion Assessment Method (CAM), help clinicians quickly recognize delirium. Criteria of the CAM tool include an acute or fluctuating change in mentation, inattention, disorganized thinking, and altered level of consciousness. Precipitating factors of delirium are numerous and include: - Medications (eg, opioids, anticholinergics) (Option 1) - Electrolyte imbalances (eg, hyponatremia) (Option 2) - Hypoxia (Option 3) - Acute infection (eg, fever, positive culture) (Options 4 and 5) - Sleep deprivation - Dehydration or malnutrition - Metabolic disorders (eg, hypoglycemia) Nursing interventions include treating the underlying cause as prescribed to resolve delirium (eg, antibiotics, supplemental oxygen), maintaining a safe environment (eg, continuous monitoring, room near the nurses' station, bed alarm), reorienting the client frequently, promoting a regular sleep cycle, providing familiar items from home, and encouraging family and friends to stay with the client. Educational objective: The Confusion Assessment Method is an assessment tool that helps clinicians quickly recognize delirium, which is characterized by an acute or fluctuating change in mental status and is often caused by medications (eg, opioids) or an underlying medical condition (eg, hypoxia, electrolyte imbalances, infection).

The nurse is caring for a client diagnosed with Guillain-Barré syndrome (GBS) after a recent gastrointestinal (GI) illness. Monitoring for which of the following is a nursing care priority for this client? 1. Diaphoresis with facial flushing 2. Hypoactive or absent bowel sounds 3. Inability to cough or lift the head 4. Warm, tender, and swollen leg

3 GBS is an acute, immune-mediated polyneuropathy that is most often accompanied by ascending muscle weakness and absent deep-tendon reflexes. Many clients have a history of antecedent respiratory tract or GI infection. Lower-extremity weakness progresses over hours to days to involve the thorax, arms, and cranial nerves. However, neuromuscular respiratory failure is the most life-threatening complication. Early signs indicating impending respiratory failure include: Inability to cough Shallow respirations Dyspnea and hypoxia Inability to lift the head or eye brows Assessing the client's pulmonary function by serial spirometry is also recommended. Measurement of forced vital capacity (FVC) is the gold standard for assessing ventilation; a decline in FVC indicates impending respiratory arrest requiring endotracheal intubation. (Option 1) Severe autonomic dysfunction can present as diaphoresis and facial flushing. (Option 2) The client with GBS is also at risk for paralytic ileus, which is related to either immobility or nerve damage. As a result, the nurse should monitor for the presence hypoactive/absent bowel sounds. (Option 4) Clients with GBS are at risk of developing deep venous thrombosis due to lack of ambulation and should receive pharmacologic prophylaxis (heparin) and support stockings. Although symptoms in options 1, 2, and 4 represent a progressive illness and are important to communicate to the health care provider promptly, they are not the highest priority compared to impending respiratory failure. Educational objective: Respiratory distress is a potential complication of progressing paralysis in clients with Guillain-Barré syndrome. The nurse should prioritize and monitor for the presence of this complication. Measurement of serial spirometry (FVC) is the gold standard for assessing ventilation.

The client has increased intracranial pressure with cerebral edema, and mannitol is administered. Which assessment should the nurse make to evaluate if a complication from the mannitol is occurring? 1. Auscultate breath sounds to assess for crackles 2. Monitor for >50 mL/hr urine output 3. Monitor Glasgow Coma Scale increasing from 8/15 to 9/15 4. Press over the tibia to assess for pitting edema

1 Mannitol (Osmitrol) is an osmotic diuretic used to treat cerebral edema (increased intracranial pressure) and acute glaucoma. When administered, mannitol causes an increase in plasma oncotic pressure (similar to excess glucose) that draws free water from the extravascular space into the intravascular space, creating a volume expansion. This fluid, along with the drug, is excreted through the kidneys, thereby reducing cerebral edema and intracranial pressure. However, if a higher dose of mannitol is given or it accumulates (as in kidney disease), fluid overload that may cause life-threatening pulmonary edema results. An early sensitive indicator of fluid overload is new onset of crackles auscultated in the lungs. To prevent these complications, clients require frequent monitoring of serum osmolarity, input and output, serum electrolytes, and kidney function. (Option 2) Urine output would be expected to increase from the diuretic effect of mannitol. This is not a complication. (Option 3) Glasgow Coma Scale scores range from 3-15. Improved mental status (orientation, alertness) is a desired effect of treatment. (Option 4) The presence of crackles is a more sensitive sign of fluid overload than pedal edema. Furthermore, in a bedridden client, the assessment should take place at a dependent part of the body, usually the sacral area. Educational objective: Mannitol is an osmotic diuretic used to treat cerebral edema and acute glaucoma. Normal kidney function and adequate urine output are crucial while administering this medication as mannitol accumulation can result in significant volume expansion, dilutional hyponatremia, and pulmonary edema.

A client with an acute head injury cannot accurately identify the sensation felt when the nurse touches the intact skin with a cotton ball or paper clip. The nurse is aware that the deficit reflects injury to which area of the brain? (picture)

The parietal lobe of the brain integrates somatic and sensory input. Injury to the parietal lobe could result in a deficit with sensation. The nurse would verify the client's injuries and documented imaging studies to confirm that this was an expected deficit and document it accordingly. If it is a new or unexpected deficit, the nurse should inform the health care provider immediately. The frontal lobe controls higher-order processing, such as executive function and personality. Injury to the frontal lobe often results in behavioral changes. The temporal lobe integrates visual and auditory input and past experiences. Temporal lobe injury clients cannot understand verbal or written language. The occipital lobe of the brain registers visual images. Injury to the occipital lobe could result in a deficit with vision. Educational objective: The frontal lobe controls executive function and personality. The temporal lobe receives auditory input. The parietal lobe receives sensory input. The occipital lobe receives visual images.

The daughter of an 80-year-old client recently diagnosed with Alzheimer disease (AD) says to the nurse, "I guess I can anticipate getting this disease myself at some point." What is an appropriate response by the nurse? 1. "Engaging in regular exercise decreases the risk of AD." 2. "Having a family history of AD is not a risk factor." 3. "Try not to worry about this now as you can't do anything to prevent AD." 4. "You should avoid aluminum cans and cookware to prevent AD."

1 The development of Alzheimer disease (AD) is related to a combination of genetic, lifestyle, and environmental factors. Clients with AD are usually diagnosed at age ≥65. Early-onset AD is a rare form of the disease that develops before age 60 and is strongly related to genetics. Children of clients with early-onset AD have a 50% chance of developing the disease. For late-onset AD, the strongest known risk factor is advancing age. Having a first-degree relative (eg, parent, sibling) with late-onset AD also increases the risk of developing AD (Option 2). Trauma to the brain has been associated with the development of AD in the future. Brain trauma may be prevented by wearing seat belts and sports helmets and taking measures to prevent falls. Research suggests that healthy lifestyle choices (eg, smoking cessation, avoiding excessive alcohol intake, exercising regularly, participating in mentally challenging activities) reduce the risk for developing AD (Options 1 and 3). (Option 4) Research has failed to confirm that exposure to aluminum products (eg, cans, cookware, antiperspirant deodorant) is related to the development of AD. Educational objective: Research suggests that healthy lifestyle choices (eg, smoking cessation, avoiding excessive alcohol intake, exercising regularly, participating in mentally challenging activities) reduce the risk for developing Alzheimer disease.

The nurse completes a neurological examination on a client who has suffered a stroke to determine if damage has occurred to any of the cranial nerves. The nurse understands that damage has occurred to cranial nerve IX based on which assessment finding? 1. A tongue blade is used to touch the client's pharynx; gag reflex is absent 2. Only one side of the mouth moves when the client is asked to smile and frown 3. The absence of light touch and pain sensation on the left side of the client's face 4. When the client shrugs against resistance, the left shoulder is weaker than the right

1 Cranial nerves IX (glossopharyngeal) and X (vagus) are related to the movement of the pharynx and tongue. To evaluate cranial nerves IX and X, the nurse assesses for the presence of a gag reflex and symmetrical movements of the uvula and soft palate, and listens to voice quality. A tongue blade can be used to touch the posterior pharyngeal wall to assess for a gag reflex. Asking the client to say "ah" will allow assessment of the uvula and soft palate. Harsh or brassy voice quality indicates dysfunction with the vagus nerve (X) (Option 1). (Option 2) The facial nerve (VII) is assessed by observing for symmetrical movements during facial expressions (eg, smile, frown, close eyes). (Option 3) The trigeminal nerve (V) has both sensory and motor functions. The nurse assesses for equal jaw strength by palpating the masseter muscle while the client clenches the jaw. To assess sensory function, the nurse touches the client's face with the client's eyes closed to determine if sensations are equal. (Option 4) The spinal accessory nerve innervates the sternocleidomastoid and part of the trapezius muscles. The nurse applies resistance during shrugging and head turning and assesses for equal strength. Educational objective: Cranial nerves IX and X work together to create movement of the pharynx and tongue. An absent gag reflex, asymmetrical uvular and palate movement, or a change in voice quality indicates damage.

A client with blunt head injury is admitted for observation, including hourly neurologic checks. At 01:00 AM, the client reports a headache; the nurse obtains a normal neurologic assessment and administers the PRN acetaminophen. At 02:00 AM, the client appears to be sleeping. Which action should the nurse take? 1. Arouse the client and ask what the current month is 2. Document "relief apparently obtained" and recheck at 03:00 AM 3. Let the client sleep but verify respiratory rate 4. Wake the client up and check for paresthesia

1 Serial neurologic assessments are important as neurologic abnormalities are often initially subtle, making it important to note the trend. Interventions for neurologic issues are most effective when made early. A neurologic assessment includes: - Glasgow Coma Scale (GCS)—best eye, verbal, and motor responses. Best verbal response assesses orientation to person, place, and time (time is the most sensitive). - Pupils—equal, round, response to light, and accommodate (PERRLA) - Motor—strength and movement in all four extremities - Vital signs—especially any signs of Cushing's triad of bradycardia, bradypnea/abnormal breathing pattern and widening pulse pressure (the difference between systolic and diastolic blood pressure readings). The nurse is assessing for signs of increased intracranial pressure (ICP). This client is not admitted in the hospital to get a good night sleep. The client is admitted due to the need for serial neurologic assessments by a professional nurse, and that is the priority. (Option 2) Although pain relief has probably been achieved, this option does not reflect any neurologic assessment. One of the early signs of increased ICP is change in level of consciousness. This option does not assess the client's arousability. (Option 3) Checking the respiratory rate and characteristics is part of the neurologic assessment. However, this alone is insufficient for assessing a neurologic status after a blunt head injury. (Option 4) It is good to awaken the client, but paresthesia is part of a neurovascular assessment. Neurovascular assessment, commonly known as the 5 Ps, consists of paresthesia, pain, pallor, paralysis, and pulselessness. A neurovascular assessment is used when circulation is a primary concern. It is not the primary neurologic assessment needed in a closed head injury. Educational objective: Clients must be awakened for a prescribed, necessary neurologic assessment. A neurologic assessment consists of GCS, PERRLA, movement and strength of the extremities, and vital signs.

The nurse is caring for a client with an acute ischemic stroke who has a blood pressure of 178/95 mm Hg. The health care provider prescribes as-needed antihypertensives to be given if the systolic pressure is >200 mm Hg. Which action by the nurse is most appropriate? 1. Give the antihypertensive medication 2. Monitor the blood pressure 3. Notify the health care provider 4. Question the prescription

2 An ischemic stroke is a loss of brain tissue perfusion due to blockage in blood flow. Elevated blood pressure is common and permitted after a stroke and may be a compensatory mechanism to maintain cerebral perfusion distal to the area of blockage. This permissive hypertension usually autocorrects within 24-48 hours and does not require treatment unless the hypertension is extreme (systolic blood pressure >220 mm Hg or diastolic blood pressure >120 mm Hg) or contraindicated due to the presence of another illness requiring strict blood pressure control (eg, active ischemic coronary disease, heart failure, aortic dissection). A blood pressure of 178/95 mm Hg should be monitored, along with the client's other vital signs and status (Option 2). (Option 1) The elevated blood pressure may be a protective measure to ensure tissue perfusion; therefore, the antihypertensive drug should not be given unless indicated by the prescription parameters. (Options 3 and 4) The as-needed prescription is appropriate in this case, so there is no reason to question it and call the health care provider. Educational objective: Elevated blood pressure in the presence of an acute ischemic stroke is an expected finding and may be a protective measure to maintain tissue perfusion.

The nurse provides education for caregivers of a client with Alzheimer disease. Which instructions should the nurse include? Select all that apply. 1. Complete activities such as bathing and dressing as quickly as possible 2. Decrease the client's anxiety by limiting the number of choices offered 3. Redirect the client if agitated by asking for help with a task or going for a walk 4. Remember to interact with the client as an adult, regardless of childlike affect 5. Use open-ended questions when communicating with the client

2, 3, 4 Strategies for caring for clients with Alzheimer disease address progressive memory loss and declining ability to communicate, think clearly, and perform activities of daily living. Caregivers should also learn to manage clients' problematic behavior and mood swings. Therapeutic guidelines include: - Use distraction and redirection (eg, going for a walk) to manage agitation (Option 3). - Speak slowly and use simple words and yes-or-no questions. - Do not try to rationalize with the client. - Use visual cues when giving directions. - Interact with the client as an adult, even as the client regresses to childlike affect and behavior; respect client dignity by avoiding use of pet names (eg, "honey," "sweetie," "darling") (Option 4). - Break down complex activities into steps with simple instructions. - Decrease the client's anxiety by limiting the number of choices (Option 2). (Option 1) Allow plenty of time for task completion. The client cannot process information rapidly, and hurrying or rushing can cause agitation or anxiety. (Option 5) Ask questions that can be answered with yes, no, or very few words. Do not ask open-ended questions, which can overwhelm the client and cause increased stress and frustration. Educational objective: Caregivers for clients with Alzheimer disease should communicate with the client using yes-or-no questions and simple, step-by-step instructions; treat the client as an adult; limit the number of choices; and allow plenty of time for task completion. Agitated clients can be redirected with new activities (eg, going for a walk).

A client with stroke symptoms has a blood pressure of 240/124 mm Hg. The nurse prepares the prescribed nicardipine intravenous (IV) infusion solution correctly to yield 0.1 mg/mL. The nurse then administers the initial prescription to infuse at 5 mg/hr by setting the infusion pump at 50 mL/hr. What is the nurse's priority action at this time? 1. Assess hourly urinary output 2. Increase pump setting to correct administration rate to 100 mL/hr 3. Keep systolic blood pressure above 170 mm Hg 4. Monitor for a widening QT interval

3 A client with an acute stroke presentation (brain attack) requires "permissive hypertension" during the first 24-48 hours to allow for adequate perfusion through the damaged cerebral tissues. However, the blood-brain barrier is no longer intact once the blood pressure is >220/120 mm Hg. Therefore, "mild" lowering is required, usually to a systolic pressure that is not below 170 mm Hg. Nicardipine (Cardene) is a prototype of nifedipine and is a potent calcium channel blocking vasodilator. It takes effect within 1 minute of IV administration. It is essential to monitor that the blood pressure is not being lowered too quickly or too slowly as this would extend the stroke. Hypotension can occur with or without reflex tachycardia. The drug must be discontinued if hypotension or reflex tachycardia occurs. (Option 1) It is important to watch for signs of adequate perfusion to the kidneys as indicated by adequate urine output. Approximately 25% of the cardiac output goes to the kidney. However, the priority is to monitor for the rapid onset of effect that this potent antihypertensive agent has after initiation. The nurse should not wait until the urine output drops. (Option 2) The initial rate set by the nurse is correct as desired dose/dose on hand is 5 mg/hr divided by 0.1 mg/mL and equals 50 mL/hr. (Option 4) Widening of the QT interval can increase the risk of life-threatening torsades de pointes. It is most commonly seen with haloperidol (Haldol), methadone, ziprasidone (Geodon), and erythromycin. However, this is not an expected complication of nicardipine. Educational objective: Nicardipine (Cardene) is a potent calcium channel blocking vasodilator. The priority nursing intervention with IV administration is to monitor the hypotensive effects of this drug.

The nurse is caring for a client with absence seizures. The unlicensed assistive personnel (UAP) asks if the client will "shake and jerk" when having a seizure. Which response from the nurse is the most helpful? 1. "No, absence seizures can look like daydreaming or staring off into space." 2. "No, you are wrong. Don't worry about that." 3. "Yes, so please let me know if you see the client do that." 4. "You don't have to monitor the client for seizures."

1 Absence seizures typically occur in children. The presentation is classic and includes the following: - Daydreaming episodes or brief (<10 seconds) staring spells - Absence of warning and postictal phases - Absence of other forms of epileptic activity (no myoclonus or tonic-clonic activity) - Unresponsiveness during the seizure - No memory of the seizure The most helpful response by the nurse is one that corrects while educating the UAP (Option 1). The UAP may be present when a client has a seizure, and understanding of what to expect will aid client care. (Options 2 and 4) Although it is not the responsibility of the UAP to monitor the client, the UAP may witness a seizure and call for help if needed. (Option 3) Seizures may include tonic (body stiffening), clonic (muscle jerking), atonic (loss of muscle tone or "drop attack"), myoclonic (brief muscle jerk), or tonic-clonic (alternating stiffening and jerking) body motions. Absence seizures do not typically involve these body motions. Educational objective: Absence seizures are brief periods of staring; there is no evidence of tonic-clonic activity or postictal confusion. The UAP should be educated about absence seizures when involved in the care of such clients.

The nurse is caring for a client after a motor vehicle accident. The client's injuries include 2 fractured ribs and a concussion. The nurse notes which of the following as expected neurological changes for the client with a concussion? Select all that apply. 1. Asymmetrical pupillary constriction 2. Brief loss of consciousness 3. Headache 4. Loss of vision 5. Retrograde amnesia

2, 3, 5 A concussion is considered a minor traumatic brain injury and results from blunt force or an acceleration/deceleration head injury. Typical signs of concussion include: - A brief disruption in level of consciousness - Amnesia regarding the event (retrograde amnesia) - Headache These clients should be observed closely by family members and not participate in strenuous or athletic activities for 1-2 days. Rest and a light diet are encouraged during this time. (Options 1 and 4) The following manifestations indicate more serious brain injury and are not expected with simple concussion: - Worsening headaches and vomiting (indicate high intracranial pressure) - Sleepiness and/or confusion (indicate high intracranial pressure) - Visual changes - Weakness or numbness of part of the body Educational objective: Expected neurological changes with a concussion include brief loss of consciousness, retrograde amnesia, and headache. These clients should be observed closely by family members and not participate in strenuous or athletic activities for 1-2 days.

Assessment of a client with a history of stroke reveals that the client understands and follows commands but answers questions with incorrect word choices. The nurse documents the presence of which communication deficit? 1. Aphasia 2. Apraxia 3. Dysarthria 4. Dysphagia

1 Aphasia refers to impaired communication due to a neurological condition (eg, stroke, traumatic brain injury). The term aphasia is interchangeable with dysphasia, although aphasia is used more commonly. Receptive aphasia refers to impaired comprehension of speech and writing. A client with receptive aphasia may speak full sentences, but the words do not make sense. The nurse should speak clearly, ask simple "yes" or "no" questions, and use gestures and pictures to increase understanding. Expressive aphasia refers to impaired speech and writing. A client with expressive aphasia may be able to speak short phrases but will have difficulty with word choice (Option 1). The nurse should listen without interrupting and give the client time to form words. A client may have one type of aphasia or a combination of both, and the severity will vary with the individual. (Option 2) Apraxia refers to loss of the ability to perform a learned movement (eg, whistling, clapping, dressing) due to neurological impairment. (Option 3) Dysarthria is weakness of the muscles used for speech. Pronunciation and articulation are affected. Comprehension and the meaning of words are intact, but speech is difficult to understand (eg, mumble, lisp). (Option 4) Dysphagia refers to difficulty swallowing. The term dysphagia is often confused with dysphasia. Clients with motor deficits after a stroke may have dysphagia, which requires swallowing precautions to prevent aspiration. Educational objective: Aphasia refers to a neurological impairment of communication. Clients may have impaired speech and writing, impaired comprehension of words, or a combination of both.

The nurse is caring for a young adult who is considering becoming pregnant. The client expresses concern, stating, "One of my parents has Huntington disease, and I am afraid my child will get it." How should the nurse respond? 1. "Genetic counseling is recommended. You will receive a referral before you leave." 2. "Huntington disease inheritance requires both biological parents to carry the gene." 3. "There are other ways to grow your family. You should consider adoption." 4. "This disease occurs spontaneously and is not likely to affect your children."

1 Huntington disease (HD) is an incurable autosomal dominant hereditary disease that causes progressive nerve degeneration, which results in impaired movement, swallowing, speech, and cognitive abilities. Chorea (involuntary, tic-like movement) is a hallmark sign. The onset of active disease is usually at age 30-50, and death from neuromuscular and respiratory complications typically occurs within 20 years of diagnosis. HD is confirmed by genetic testing. Clients who have a parent with HD and are considering having biological children should receive genetic counseling (Option 1). Educational objective: Huntington disease is an incurable autosomal dominant disease that causes progressive nerve degeneration, which impairs movement, swallowing, speech, and cognitive abilities. Death typically occurs within 20 years. Clients who have a parent with this disease should receive genetic counseling, especially when planning to start a family.

The nurse reinforces education about safety modifications in the home for the spouse of a client diagnosed with Alzheimer disease. What instructions should the nurse include? Select all that apply. 1. Arrange furniture to allow for free movement 2. Keep frequently used items within easy reach 3. Lock doors leading to stairwells and outside areas 4. Place an identifying symbol on the bathroom door 5. Provide a dark room free of shadows for sleeping

1, 2, 3, 4 When a client with Alzheimer disease is being cared for in the home, the caregiver should be instructed regarding safety modifications to ease the burden of caregiving and promote the client's independence and dignity. Injury-prevention modifications include: - Arrange furniture to allow for free movement to prevent falls (Option 1). - Place frequently used items within easy, visible reach of the client (Option 2). - Place locks on stairwells and outside doors to decrease the client's risk of falls and becoming lost during periods of wandering (Option 3). - Label the doors to the bathroom and other commonly used rooms to assist with environment interpretation and promote independent functioning (Option 4). (Option 5) Providing a night light in the sleeping area can prevent falls, aid in orientation, and decrease illusions. Educational objective: Caregivers of clients with Alzheimer disease should be taught safety modifications for the home, such as placing frequently used items within reach, arranging furniture to allow for free movement, labeling doors to commonly used rooms, providing a night light, and locking stairwell and outside doors.

The nurse is giving a presentation at a community health event. The nurse should provide which instruction on how to prevent botulism? 1. Boil water if unsure of its source 2. Discard canned food with a bulging end 3. Keep milk cold 4. Wash hands

2 Botulism is caused by the gastrointestinal absorption of the neurotoxin produced by Clostridium botulinum. The neurotoxin blocks acetylcholine at the neuromuscular junction, resulting in muscle paralysis. - The organism is found in the soil and can grow in any food contaminated with the spores. - Manifestations include descending flaccid paralysis (starting from the face), dysphagia, and constipation (smooth muscle paralysis). - The main source is improperly canned or stored food. A metal can's swollen/bulging end can be caused by the gases from C botulinum and should be discarded. - The infant form of botulism can occur in children under age 1 year if they eat honey, particularly raw (wild) honey. The immature gut system in these children makes them more susceptible. (Option 1) Contaminated water is boiled to prevent infestation with Giardia, which can cause gastrointestinal disease but is not related to botulism. (Option 3) Keeping dairy at room temperature can cause it to spoil, which would then cause gastroenteritis if ingested. Most serious illnesses are prevented through pasteurization. This is not related to botulism. (Option 4) Escherichia coli infections result from ingestion of food or water that is contaminated by feces. This can be related to improper handwashing or undercooking meat and is not related to botulism. Educational objective: Botulism is a result of ingesting improperly canned or stored food. Food in a can with a bulging end should not be used. Children under age 1 year should not be given honey as their immature gut system makes them prone to developing infant botulism.

A client with a T4 spinal cord injury has a severe throbbing headache and appears flushed and diaphoretic. Which priority interventions should the nurse perform? Select all that apply. 1. Administer an analgesic as needed 2. Determine if there is bladder distention 3. Measure the client's blood pressure 4. Place the client in the Sims' position 5. Remove constrictive clothing

2, 3, 5 Clients with a high (T6 or above) spinal cord injury are at risk for autonomic dysreflexia (autonomic hyperreflexia). It is an uncompensated sympathetic nervous system stimulation. Classic signs include hypertension (up to 300 mm Hg systolic), throbbing headache, diaphoresis above the level of injury, bradycardia (30-40/min), piloerection ("goose bumps"), flushing, and nausea. This is a life-threatening condition that requires immediate intervention to prevent complications (eg, hypertensive stroke, seizures). Clients with a spinal cord injury should have their blood pressure checked when they report a headache (Option 3). The most common cause of autonomic dysreflexia is bladder irritation due to distention. The client needs to be catheterized or the possibility of a kink in the existing catheter must be assessed (Option 2). Bowel impaction can also be a cause; a digital rectal examination should be performed. Constrictive clothing should be removed to decrease skin stimulation (Option 5). The primary health care provider should be notified. An alpha-adrenergic blocker or an arteriolar vasodilator (eg, nifedipine) may be prescribed. (Option 1) Headaches associated with autonomic dysreflexia are typically due to severe hypertension and often resolve after blood pressure has been treated. (Option 4) The client should have the head of the bed elevated 45 degrees or high Fowler's to lower blood pressure. The Sims' position is flat and side-lying. Educational objective:Autonomic dysreflexia is a life-threatening condition in a client with high spinal cord injury. Classic signs/symptoms include severe hypertension, throbbing headache, diaphoresis, bradycardia, flushing, and piloerection. Emergency treatment includes correcting the cause (check bowel or bladder distention), removing tight clothing, and raising the head of the bed

The nurse is providing care for a client with Alzheimer disease who often becomes angry and agitated 20 minutes or more after eating. The client accuses the nurse of not providing food, saying, "I'm hungry. You didn't feed me." The nurse should take which action? 1. Give the client gentle reminders that the client has already eaten 2. Say that the client can have a snack in a couple of hours 3. Serve the client half of the meal initially and offer the other half later 4. Take a picture of the client having a meal and show it when the client becomes upset

3 Most clients with Alzheimer disease experience eating and nutritional problems throughout the course of their disease. During the earlier stages, it is common for clients to forget that they have eaten recently. The best approach is for caregivers to give clients something to eat when they say they are hungry. Smaller meals throughout the day, along with low-calorie snacks, are effective strategies for clients who forget that they have eaten. (Option 1) Reality orientation has been recommended in the past as a way to deal with confusion (eg, dementia, Alzheimer disease), but research has shown that it may cause anxiety and distress. Validation therapy is a newer and more therapeutic approach that validates and accepts the client's reality. (Option 2) Offering to provide a snack later does not address the client's stated need to eat now. Delay in giving food will only further increase the client's anger and frustration. (Option 4) Showing a picture of the client having a meal is confrontational and will have no meaning to the client. Educational objective: Clients with Alzheimer disease experience eating and nutritional problems throughout the course of the disease. During the earlier stages, it is common for them to forget that they have eaten recently. The best approach is for caregivers to give clients something to eat when they say they are hungry.

A client comes to the emergency department with diplopia and recent onset of nausea. Which statement by the client would indicate to the nurse that this is an emergency? 1. "I am very tired, and it's hard for me to keep my eyes open." 2. "I don't feel good, and I want to be seen." 3. "I have not taken my blood pressure medicine in over a week." 4. "I have the worst headache I've ever had in my life."

4 A ruptured cerebral aneurysm is a surgical emergency with a high mortality rate. Cerebral aneurysms are usually asymptomatic unless they rupture; they are often called "silent killers" as they may go undetected for many years before rupturing without warning signs. The distinctive description of a cerebral aneurysm rupture is the abrupt onset of "the worst headache of my life" that is different from previous headaches (including migraines). Immediate evaluation for a possible ruptured aneurysm is critical for any client experiencing a severe headache with changes in or loss of consciousness, neurologic deficits, diplopia, seizures, vomiting, or a stiff neck. Early identification and prompt surgical intervention help increase the chance for survival. (Options 1, 2, and 3) A change in level of consciousness, increased blood pressure, or a feeling of illness should be investigated but alone does not indicate an emergency. Educational objective: Sudden onset of a severe headache described as "the worst headache of my life" is characteristic of a ruptured cerebral aneurysm and should be treated as an emergency.

A client with a C3 spinal cord injury has a headache and nausea. The client's blood pressure is 170/100 mm Hg. How should the nurse respond initially? 1. Administer PRN analgesic medication 2. Administer PRN antihypertensive medication 3. Lower the head of the bed 4. Palpate the client's bladder

4 Autonomic dysreflexia is an acute, life-threatening response to noxious stimuli, which clients with spinal cord injuries above T6 are unable to feel. Signs and symptoms include hypertension, bradycardia, a pounding headache, diaphoresis, and nausea. It is essential that the nurse assess for and remove noxious stimuli to prevent a stroke (Option 4). Noxious stimuli may include: - Bladder distention (eg, obstructed urinary catheter, neurogenic bladder) - Fecal impaction - Tight clothing (eg, shoelaces, waistbands) (Options 1 and 2) Hypertension, headache, and nausea due to uncontrolled sympathetic activity will resolve once the cause is identified and removed. (Option 3) Lowering the head of the bed would increase blood pressure. The head of the bed should be raised to lower the blood pressure. Educational objective: Autonomic dysreflexia is an acute, life-threatening response to noxious stimuli (eg, bladder distention, tight clothing) in clients with spinal cord injuries above T6. Signs and symptoms include hypertension, bradycardia, a pounding headache, diaphoresis, and nausea. The nurse must immediately identify and remove noxious stimuli to prevent a stroke and resolve symptoms.

The emergency department nurse assesses a client involved in a motor vehicle accident who sustained a coup-contrecoup head injury. Which assessment finding is consistent with injury to the occipital lobe? 1. Decreased rate and depth of respirations 2. Deficits in visual perception 3. Expressive aphasia 4. Inability to recognize touch

2 A coup-contrecoup head injury occurs when the head strikes an object and the brain receives an injury under the area of impact (coup), after which it rebounds to the opposite side of the skull and sustains injury on that side as well (contrecoup). This type of injury is common in motor vehicle accidents and shaken baby syndrome. Visual processing occurs in the occipital lobe. (Option 1) The rate and depth of respirations are regulated by the medulla oblongata at the base of the brainstem. (Option 3) Expressive aphasia, the inability to express spoken words, occurs after a transient ischemic attack or stroke. This will occur if the frontal lobe (Broca aphasia) or temporal lobe (Wernicke aphasia) is injured. (Option 4) Inability to recognize being touched is indicative of injury to the parietal lobe of the brain. Educational objective: Coup-contrecoup head injuries are common in motor vehicle accidents and shaken baby syndrome. Damage to the occipital lobe of the brain during coup-contrecoup head injury will result in visual disturbances.

The nurse is caring for a client after a lumbar puncture (spinal tap). Which client assessment is most concerning and requires a nursing response? 1. Consumes 600 mL liquid over 4 hours 2. Insertion site dressing saturated with clear fluid 3. Observed lying in the right-sided Sim's position 4. Reports a headache rated 6/10

2 A lumbar puncture involves removing a sample of cerebrospinal fluid through a needle inserted between vertebrae. Elevated intracranial pressure is a contraindication to performing a lumbar puncture. The client is placed in the fetal position or sitting and leaning over a table. Continued leaking fluid indicates that the site did not seal off and a blood patch (autologous blood into the epidural space) is required. (Option 1) Fluids are encouraged to help replace the cerebrospinal fluid. (Option 3) The client should lie flat for at least 4 hours. The prone or supine position is recommended to help prevent a headache. (Option 4) Up to 5%-30% of clients have the common complication of headache. It is thought to be a result of leakage of fluid through the dural puncture site. The symptom is treated and is normally self-limiting. Educational objective: After a lumbar puncture, cerebrospinal fluid leakage from the puncture site requires health care provider notification for a blood patch. A headache after the procedure is an expected finding. The client should lie flat and increase fluid intake afterwards.

The home health nurse teaches an elderly client with dysphagia some strategies to help limit repeated hospitalizations for aspiration pneumonia. Which statement indicates that the client needs further teaching? 1. "I have to remember to raise my chin slightly upward when I swallow." 2. "I have to remember to swallow 2 times before taking another bite of food." 3. "I should avoid taking over-the-counter cold medications when I'm sick." 4. "I should sit upright for at least 30-40 minutes after I eat."

1 Dysphagia increases the risk for aspiration of oropharyngeal secretions, gastric content, food, and/or fluid into the lungs. Aspiration of foreign material into the lungs increases the risk for developing aspiration pneumonia. Interventions to help decrease aspiration and resulting aspiration pneumonia in susceptible clients (eg, elderly, neurologic dysfunction, decreased cough or gag reflexes, decreased immunity, chronic disease), include the following: - Swallowing 2 times before taking another bite of food. This clears food from the pharynx. - Thickening liquids to assist swallowing - Avoiding over-the-counter cold medications. Antihistamine cold preparation medications also have some anticholinergic properties, such as causing drowsiness, decreasing saliva (xerostomia) production, and making the mouth dry. Saliva is a lubricant, and it helps bind food together to facilitate swallowing. - Sitting upright for at least 30-40 minutes after meals. This uses gravity to move food or fluid through the alimentary tract, decreases gastroesophageal reflux, and helps decrease risk for aspiration. - Brushing teeth and using antiseptic mouthwash before and after meals. This reduces the bacterial count before eating because bacteria as well as food can be aspirated. After-meal use removes particles of food that can be aspirated later. - Smoking cessation. Smoking decreases mucociliary clearance and increases bacterial count in the mouth. (Option 1) Positioning the chin slightly downward toward the neck (chin-tuck) when swallowing can be effective in some clients with dysphagia due to its facilitating closure of the epiglottis to help prevent tracheal aspiration. Educational objective: Teaching clients who are susceptible to aspiration about swallowing techniques, positioning, avoidance of over-the-counter cold preparation medications (cause drowsiness and dry mouth), oral care, and smoking cessation can decrease the risk for aspiration pneumonia

The nurse assesses several clients using the Glasgow Coma Scale. Which scenario best demonstrates a correct application of this scale? 1. The nurse applies pressure to the nail bed, and the client tries to push the nurse's hand away. The nurse scores motor response as "localization of pain." 2. The nurse asks the client what day it is and the client says "banana." The nurse scores verbal response as "confused." 3. The nurse speaks with client and then the client's eyes open. The nurse scores eye opening as "spontaneous." 4. The nurse walks in the room and the client states "Hi honey. How are you?" The nurse scores verbal response as "oriented."

1 The Glasgow Coma Scale is used to determine level of consciousness. The nurse follows a standardized assessment to determine the score of the client's eye opening response, verbal response, and ability to obey commands through a motor response. For the best motor response score, the nurse first verbally asks the client to obey a command. If there is no response, the nurse next uses noxious stimuli (eg, nail bed pressure) and records the physical response. If the client tries to remove the painful stimulus, it is recorded as "localizing" or moving toward the pain; whereas if the client retracts from the stimulus, it is recorded as "withdrawal" (Option 1). (Option 2) To ensure an accurate score in the verbal response category, the nurse must differentiate if the client is confused (eg, answers "1955" when asked the year) or if a client uses inappropriate words. (Option 3) To ensure an accurate eye opening score, the nurse must determine whether the client's eyes open spontaneously (eg, no prompting) or if a stimulus (eg, sound, pain) is needed. (Option 4) A social, verbal client is not necessarily oriented. The nurse must assess orientation by specifically asking clients to state their name, the time, and their location. Educational objective: The Glasgow Coma Scale is used to determine level of consciousness. The nurse follows a standardized assessment to determine the score of the client's eye opening response, verbal response, and ability to obey commands through a motor response.

The emergency department nurse receives several prescriptions for a client who was found unresponsive after drinking beer and consuming unidentified pills. Which prescription should the nurse implement first? Click on the exhibit button for additional information. Vital signs= Temperature 96.4 F (35.8 C) Blood pressure90/62 mm Hg Heart rate 53/min Respirations 6/min O2 saturation or SpO2 92% 1. Administer IV push naloxone once now 2. Draw specimen for blood alcohol content testing STAT 3. Initiate continuous lactated Ringer solution infusion 4. Obtain urine sample for drug abuse screening ASAP

1 The goals of emergency care for the client with suspected substance abuse who exhibits signs of central nervous system depression (eg, altered level of consciousness, bradypnea, hypotension, bradycardia) are to promote adequate ventilation and oxygenation and preserve hemodynamic stability. Interventions are prioritized according to the ABCs (ie, airway, breathing, circulation). Initial actions involve maintaining patency of the client's airway, including appropriate positioning, oropharyngeal suctioning, and artificial airway placement (if needed). Respiratory depression occurring after the ingestion of an unknown substance (eg, depressants [opioids, benzodiazepines, barbiturates]) should initially be treated with administration of reversal agents (eg, naloxone, flumazenil). Naloxone rapidly reverses the effects of opioids and may restore spontaneous respiration and normal ventilatory pattern, averting initiation of mechanical ventilation, the possibility of respiratory arrest, and death (Option 1). (Options 2 and 4) Obtaining blood and urine for toxicology screening assists in guiding care decisions but should occur after interventions that support the client's airway, breathing, and circulation. (Option 3) Administration of IV fluids to support blood pressure and prevent dehydration should be performed after securing the client's airway and supporting effective breathing. Educational objective: Nurses providing emergency care to clients with suspected substance abuse who exhibit signs of central nervous system depression (eg, bradypnea, bradycardia) prioritize interventions according to the ABCs (ie, airway, breathing, circulation). Administration of naloxone is a priority action in the setting of respiratory depression from an unknown substance because it rapidly reverses the depressant effects of opioids.

A client with a ventriculoperitoneal shunt has a dazed appearance and grunting and has not responded to the caregiver for 10 minutes. Status epilepticus is suspected. Which nursing intervention should be performed first? 1. Administer rectal diazepam 2. Assess for neck stiffness and Brudzinski sign 3. Draw blood for laboratory studies 4. Transport the client to CT for assessment of shunt malfunction

1 This client is in status epilepticus, a serious and life-threatening emergency in which a client has been seizing for 5 minutes or longer. Grunting and a dazed appearance are 2 common signs. A client with hydrocephalus (abnormal collection of cerebrospinal fluid in the head) and a ventriculoperitoneal (VP) shunt is at a higher risk for seizures. Stopping seizure activity is the first nursing priority. IV benzodiazepines (diazepam or lorazepam) are used acutely to control seizures. However, rectal diazepam is often prescribed when the IV form is unavailable or problematic. Parents often get prescriptions for rectal diazepam and are advised to administer a dose before bringing a child to the emergency department. (Option 2) Stopping the status epilepticus is a priority over determining its cause through a neurologic assessment. Quickly obtaining the oxygen saturation level and managing the airway are priority assessments. (Option 3) Blood draw is needed for laboratory studies but is not a priority over stopping the seizure. (Option 4) A VP shunt drains excess fluid in the brain down to the abdomen, where it is absorbed by the body. A CT scan can accurately assess shunt malfunction. Any malfunction would need to be treated promptly to prevent future seizures and damage. Finding the cause of the seizure is important and should be done as soon as seizing has stopped. Educational objective: Status epilepticus is a serious condition that could result in brain damage and death. Quickly stopping the seizure is the first nursing priority as long as there is an adequate airway and the client is breathing. IV or rectal benzodiazepines (lorazepam or diazepam) are used to rapidly control seizures.

The nurse educates the caregiver of a client with Alzheimer disease about maintaining the client's safety. Current symptoms include occasional confusion and wandering. Which of the following responses by the caregiver show correct understanding? Select all that apply. 1. "Grab bars should be installed in the shower and beside the toilet." 2. "I will place a safe return bracelet on the client's wrist." 3. "Keyed deadbolts should be placed on all exterior doors." 4. "Medications will be placed in a weekly pill dispenser." 5. "Throw rugs and clutter will be removed from the floors."

1, 2, 3, 5 Alzheimer disease (AD) is a form of dementia that causes progressive decline of cognitive and physical abilities. The nurse should educate the client/caregiver to prepare for current and future safety needs. Interventions evolve to meet client needs at each stage of disease progression. Safety promotion for the client with moderate AD includes: - Keyed deadbolts (with keys removed) and close supervision to provide a controlled environment for wandering (Option 3) - Medical identification/location devices (eg, bracelets, shoe inserts) in case the client wanders outside the designated area (Option 2) - Decreased water heater temperature and "hot" and "cold" labels on faucets to prevent burns - Household hazards (eg, gas appliances, rugs, toxic chemicals) removed to prevent injury (Option 5) - Grab bars installed in showers and tubs (Option 1) (Option 4) All medications should be out of the client's reach or locked away. A confused person may not remember the day of the week and take more or less medication than prescribed. Educational objective: For clients with moderate Alzheimer disease, caregivers should provide a controlled environment for safe wandering (eg, throw rugs and clutter removed, exterior doors secured), and the client should wear an identification/location device (eg, bracelet). All medications should be out of reach or locked away. Hazards (eg, gas appliances, rugs, toxic chemicals) should be removed. Grab bars should be installed in showers and tubs.

The nurse is caring for a client at risk for aspiration pneumonia due to a stroke. What nursing actions help prevent this potential complication during hospitalization? Select all that apply. 1. Add a thickening agent to the fluids 2. Avoid administering sedating medications before meals 3. Place the client in an upright position during meals 4. Restrict visitors who show signs of illness 5. Teach the client to flex the neck while swallowing

1, 2, 3, 5 Aspiration pneumonia develops when aspirated material (eg, food, emesis, gastric reflux) causes an inflammatory response and provides a medium for bacterial growth. At-risk conditions include cognitive changes (eg, dementia, head injury, stroke, sedation), difficulty swallowing, compromised gag reflex, and tube feeding. Aspiration-prevention measures include: - Thicken liquids (eg, to nectar or honey consistency) for clients with dysphagia; thin liquids are more difficult to control when swallowing (Option 1). - Ensure that the client is fully awake before eating. The nurse should time the administration of sedating medications (eg, opioids, benzodiazepines) to avoid sedation during meals (Option 2). - Elevate the head of the bed to 90 degrees during and for 30 minutes after meals, and never place the head of the bed lower than 30 degrees (Option 3). - Encourage clients to facilitate swallowing by flexing the neck (chin to chest) (Option 5). - Administer prescribed antiemetics (eg, ondansetron) as needed to prevent vomiting. - Monitor for coughing, gagging, and pocketing food. (Option 4) Performing strict handwashing and limiting sick visitors are important infection-control measures; however, they do not prevent noninfectious aspiration pneumonia. Educational objective: Measures for preventing aspiration pneumonia include administering medications to prevent vomiting, avoiding mealtime sedation, maintaining head-of-bed elevation at 30 degrees or more (90 degrees during and 30 minutes after meals), and encouraging neck flexion while swallowing. Clients with dysphagia should receive thickened liquids and be monitored for coughing, gagging, and pocketing food.

The emergency department nurse is assessing a client brought in after a car accident in which the client's head hit the steering column. Which assessment findings would indicate that the triage nurse should apply spinal immobilization? Select all that apply. 1. Breath smells of alcohol 2. Client disoriented to place 3. Client reports eyes burning 4. History of multiple sclerosis 5. Point tenderness over spine

1, 2, 5 Spinal immobilization is not a benign procedure. An acronym to help determine the need for spinal immobilization is NSAIDs: N - Neurological examination. Focal deficits include numbness and decreased strength. S - Significant traumatic mechanism of injury A - Alertness. The client may be disoriented or have an altered level of consciousness (Option 2). I - Intoxication. The client could have impaired decision-making ability or lack awareness of pain (Option 1). D - Distracting injury. Another significant injury could distract the client from spinal pain. S - Spinal examination. Point tenderness over the spine or neck pain on movement (if there is no midline tenderness) may be present (Option 5). (Option 3) The sensation of burning eyes could be related to many issues and does not necessarily have a direct correlation to spinal trauma. (Option 4) There is no direct correlation of multiple sclerosis (autoimmune progressive nerve demyelinization) with the need for spinal immobilization. Educational objective: Indications for spinal immobilization include abnormal neurological findings, significant mechanism of injury, change in orientation or level of consciousness, intoxication, distracting injury, and point tenderness over the spine.

The nurse is caring for a client with Bell palsy. Which of the following assessment findings does the nurse expect? Select all that apply. 1. Change in lacrimation on the affected side. 2. Electric shock-like pain in the lips and gums. 3. Flattening of the nasolabial fold. 4. Inability to smile symmetrically. 5. Severe pain along the cheekbone.

1, 3, 4 Bell palsy is peripheral, unilateral facial paralysis characterized by inflammation of the facial nerve (cranial nerve VII) in the absence of a stroke or other causative agent/disease. Paralysis of the motor fibers innervating the facial muscles results in flaccidity on the affected side. Manifestations of Bell palsy include: - Inability to completely close the eye on the affected side - Alteration in tear production (eg, decreased tearing with extreme dryness, excessive tearing) due to weakness of the lower eyelid muscle (Option 1) - Flattening of the nasolabial fold on the side of the paralysis (Option 3) - Inability to smile or frown symmetrically (Option 4) - Alteration in the sensory fibers can cause loss of taste on the anterior two-thirds of the tongue. (Options 2 and 5) Electric shock-like pain in the lips and gums and severe pain along the cheekbone are symptoms of trigeminal neuralgia (cranial nerve V). With Bell palsy, the trigeminal nerve may become hypersensitive and cause facial pain, but this is uncommon and typically more indicative of trigeminal neuralgia. Educational objective: Bell palsy is unilateral facial paralysis due to inflammation of the facial nerve that is characterized by inability to close the affected eye completely, changes in tear production, facial droop, and asymmetrical smile or frown.

A client is diagnosed with right-sided Bell's palsy. What instructions should the nurse give this client for care at home? Select all that apply. 1. Apply a patch to the right eye at night 2. Avoid driving 3. Chew on the left side 4. Maintain meticulous oral hygiene 5. Use a cane on the left side

1, 3, 4 Bell's palsy is an inflammation of cranial nerve VII (facial) that causes motor and sensory alterations. Clients are usually managed as outpatients, with corticosteroids to reduce inflammation, and taught eye/oral care. In Bell's palsy, the eyelids do not close properly. This may result in eye dryness and risk of corneal abrasions. However, weakness of the lower eyelid may cause excessive tearing due to overflow in some clients. Facial muscle weakness results in poor chewing and food retention. Client teaching should include the following: 1) Eye care: Use glasses during the day; wear a patch (or tape the eyelids) at night to protect the exposed eye. Use artificial tears during the day as needed to prevent excess drying of the cornea (Option 1). 2) Oral care: Chew on the unaffected side to prevent food trapping; a soft diet is recommended. Maintain good oral hygiene after every meal to prevent problems from accumulated residual food (eg, parotitis, dental caries) (Options 3 and 4). (Options 2 and 5) Vision, balance, consciousness, and extremity motor function are not impaired with Bell's palsy. Educational objective: Bell's palsy is an inflammation of cranial nerve VII (facial) that results in facial muscle weakness and inability to close the eyelids. Eye care (patch at night, artificial tears as needed) and oral care (eating on the unaffected side, oral hygiene after meals) are vital for these clients.

The nurse is caring for a client who had a stroke two weeks ago and has moderate receptive aphasia. Which interventions should the nurse include in the plan of care to help the client follow simple commands regarding activities of daily living (ADL)? Select all that apply. 1. Ask simple questions that require "yes" or "no" answers 2. If the client becomes frustrated, seek a different care provider to complete ADL 3. Remain calm and allow the client time to understand each instruction 4. Show the client pictures of ADL (eg, shower, toilet, and toothbrush) or use gestures 5. Speak slowly but loudly while looking directly at the client

1, 3, 4 Receptive aphasia refers to impairment or loss of language comprehension (ie, speech, reading) that is caused by a neurological condition (eg, stroke, traumatic brain injury). The terms "aphasia" and "dysphasia" can be used interchangeably as both refer to impaired communication; however, "aphasia" is more commonly used. When assisting a client with receptive aphasia to complete activities of daily living, the nurse should avoid completing tasks for the client and should instead encourage independence using appropriate communication techniques. Appropriate interventions to aid communication include: - Ask short, simple, "yes" or "no" questions (Option 1). - Use gestures or pictures (eg, communication board) to demonstrate activities (Option 4). - Remain patient and calm, allowing the client time to understand each instruction (Option 3). (Option 2) Clients with aphasia often become frustrated due to inability to communicate effectively. Frustration does not result from the nurse's care, so reassigning the client to a different care provider is not an effective solution. (Option 5) Eye contact is important in all communication, but raising the voice will not help. Speaking loudly will not improve comprehension and may increase anxiety and confusion. Educational objective: Receptive aphasia refers to impairment or loss of language comprehension. Appropriate interventions to aid communication include asking short, simple, "yes" or "no" questions; using hand gestures or pictures to demonstrate activities; and patiently allowing the client time to understand each instruction.

The nurse assesses a newly admitted adult client on a neurological inpatient unit. Which assessment findings require immediate follow-up by the nurse? Select all that apply. 1. Cannot flex the chin toward the chest 2. Eyes move in opposite direction of head when head is turned to side 3. New onset of right arm drift 4. Pupils 8 mm in diameter bilaterally 5. Toes point downward when sole of foot is stimulated

1, 3, 4 The nurse performs a neurological assessment to evaluate for changes in motor and sensory functions by assessing pupils, cranial nerves, and neuromuscular functioning. The neck should be supple and able to be flexed toward the chest. Nuchal rigidity requires follow-up due to possible meningeal irritation related to infection (eg, meningitis) (Option 1). A new-onset unilateral drift of a limb could indicate a stroke. The nurse assesses for other strokelike symptoms (eg, changing level of consciousness, asymmetrical smile, garbled speech), activates the facility stroke protocol, and notifies the health care provider accordingly (Option 3). Normal pupils are 3-5 mm in diameter. Pupil dilation can be the result of medication use or neurological causes (eg, increased intracranial pressure, brain herniation) (Option 4). (Option 2) Oculocephalic reflex (doll's eyes) is an expected finding indicating an intact brainstem. It is tested by rotating the head and watching for the eyes to move simultaneously in the opposite direction. The test is not performed if spinal trauma is suspected. (Option 5) The normal finding in adults is an absent Babinski reflex (ie, toes point downward with stimulus to the sole). The presence of Babinski reflex (ie, toes fan outward and upward with stimuli) is expected in infants up to age 1, but in an adult may indicate a brain or spinal cord lesion. Educational objective: Abnormal neurological assessments include nuchal rigidity; new-onset unilateral drift of a limb; pupils <3 mm or >5 mm in diameter; absent oculocephalic reflex; and presence of Babinski reflex in an adult client.

A hospitalized client develops acute hemorrhagic stroke and is transferred to the intensive care unit. Which nursing interventions should be included in the plan of care? Select all that apply. 1. Administer PRN stool softeners daily 2. Administer scheduled enoxaparin injection 3. Implement seizure precautions 4. Keep client NPO until swallow screen is performed 5. Perform frequent neurological assessments

1, 3, 4, 5 A hemorrhagic stroke occurs when a blood vessel ruptures in the brain and causes bleeding into the brain tissue or subarachnoid space. Seizure activity may occur due to increased intracranial pressure (ICP) (Option 3). During the acute phase, a client may develop dysphagia. To prevent aspiration, the client must remain NPO until a swallow function screen reveals no deficits (Option 4). The nurse should perform neurological assessments (eg, level of consciousness, pupillary response) at regular intervals and report any acute changes (Option 5). Preventing activities that increase ICP or blood pressure will minimize further bleeding. The nurse should: - Reduce stimulation, maintain a quiet and dimly lit environment, limit visitors - Administer stool softeners to reduce strain during bowel movements (Option 1) - Reduce exertion, maintain strict bed rest, assist with activities of daily living - Maintain head in midline position to improve jugular venous return to the heart (Option 2) Enoxaparin is an anticoagulant used to prevent venous thromboembolism (VTE). Anticoagulants are contraindicated in clients with hemorrhagic stroke; the nurse should question any prescriptions that increase risk for bleeding. A client with hemorrhagic stroke should instead receive nonpharmacologic interventions (eg, compression stockings) to prevent VTE. Educational objective: A hemorrhagic stroke occurs when a blood vessel ruptures in the brain and causes bleeding. The nurse should perform frequent neurological assessments, keep the client NPO, maintain seizure precautions and strict bed rest, and limit any activity that may increase bleeding (eg, anticoagulant administration) or intracranial pressure (eg, stimulation, straining during bowel movements).

A client is admitted to the hospital with an exacerbation of myasthenia gravis. What are the appropriate nursing actions? Select all that apply. 1. Administer an anticholinesterase drug AC 2. Anticipate a need for an anticholinergic drug 3. Develop a bladder training schedule 4. Encourage semi-solid food consumption 5. Teach the necessity for annual flu vaccination

1, 4, 5 Myasthenia gravis is an autoimmune disease involving a decreased number of acetylcholine receptors at the neuromuscular junction. As a result, there is fluctuating weakness of skeletal muscles, most often presented as ptosis/diplopia, bulbar signs (difficulty speaking or swallowing), and difficulty breathing. Muscles are stronger in the morning and become weaker with the day's activity as the supply of available acetylcholine is depleted. Treatment includes anticholinesterase drugs (pyridostigmine [Mestinon]) that are administered before meals so that the client's ability to swallow is strongest during the meal (Option 1). Semi-solid foods (easily-chewed foods) are preferred over solid foods (to avoid stressing muscles involved in chewing and swallowing) or liquids (aspiration risk) (Option 4). All clients with a serious chronic co-morbidity should receive the annual flu vaccine (also the pneumonia vaccine if appropriate) as they are more likely to have a negative outcome if the illness is contracted. It is especially important in clients with myasthenia gravis as the flu (or pneumonia) would tax the already compromised respiratory muscles (Option 5). (Option 2) An anticholinergic drug, such as atropine, is used for treatment in a cholinergic crisis (eg, the medication is too high or there is excess acetylcholine). The need would not be anticipated during a myasthenic crisis (eg, exacerbation of myasthenia gravis), which is usually a result of too little medication related to noncompliance, illness, or surgery. (Option 3) The skeletal muscles are involved in myasthenia gravis; dysfunction of the reflexes or central nervous system affects bowel and bladder control. This issue is classic with multiple sclerosis. Educational objective: Myasthenia gravis involves reduction of acetylcholine receptors in the skeletal muscles; this decreases the strength of muscles used for eye and eyelid movements, speaking, swallowing, and breathing. Treatment includes administration of anticholinesterase drugs before meals, easily-chewed foods, and appropriate vaccinations.

The nurse is caring for a client admitted for a seizure disorder. The nurse witnesses the client having a tonic-clonic seizure with increasing salivation. Which actions should the nurse take? Select all that apply. 1. Call for help 2. Hold down the client's arms 3. Insert a tongue depressor to move the tongue 4. Prepare for suctioning 5. Turn the client on the side

1, 4, 5 Safety is the immediate priority in a client experiencing a seizure. Nursing interventions include: - Remain at the client's bedside while noting duration and symptoms of the seizure - Call for help so that other team members can assist with care of the client (Option 1) - Protect client from hitting hard surfaces by padding the side rails - Turn client on the side if possible to allow for drainage of secretions and prevent the tongue from occluding the airway (Option 5) - Loosen clothing around the neck and chest to promote ventilation - Use suction equipment after the seizure subsides as needed to maintain a patent airway (Option 4) The client should not be restrained as this could cause an injury (Option 2). Oral airways should be kept at the bedside for postictal airway management and recovery, but during an active seizure it is dangerous to attempt to insert anything in the client's mouth, especially if the teeth are clenched (Option 3). Educational objective: During an active seizure, the nurse should call for additional help, turn the client on the side if possible, and have suction equipment ready to clear any excessive secretions that may block the airway. The nurse should not restrain the client or force anything into the client's mouth.

An adult client with altered mental status and fever has suspected bacterial meningitis with sepsis. Blood pressure is 80/60 mm Hg. Which prescribed intervention should the nurse implement first? 1. Administer IV antibiotics 2. Infuse bolus of IV normal saline 3. Prepare to assist with lumbar puncture 4. Transport client for head CT scan

2 Meningitis is an inflammation of the meninges covering the brain and spinal cord. The key clinical manifestations of bacterial meningitis include fever, severe headache, nausea/vomiting, and nuchal rigidity. Other symptoms include photophobia, altered mental status, and other signs of increased intracranial pressure (ICP). In a hypotensive client with sepsis, the priority of care is fluid resuscitation to increase the client's blood pressure (Option 2). In addition to IV fluid administration, interventions and prescriptions for a client with sepsis and meningitis may include: - Administer vasopressors. - Obtain relevant labs and blood cultures prior to administering antibiotics. - Administer empiric antibiotics, preferably within 30 minutes of admission (Option 1). This client will continue to decline without antibiotic therapy. - Prior to a lumbar puncture (LP), obtain a head CT scan as increased ICP or mass lesions may contraindicate a LP due to the risk of brain herniation (Option 4). - Assist with a LP for cerebrospinal fluid (CSF) examination and cultures (Option 3). CSF is usually purulent and turbid in clients with bacterial meningitis. CSF cultures will allow for targeted antibiotic therapy. Educational objective: For bacterial meningitis with sepsis, fluid resuscitation is the priority. Blood cultures should be drawn before starting antibiotics. After a head CT scan is performed to rule out increased intracranial pressure and mass lesions, cerebrospinal fluid cultures should be drawn via lumbar puncture.

A client was struck on the head by a baseball bat during a robbery attempt. The nurse gives this report to the oncoming nurse at shift change and conveys that the client's current Glasgow Coma Scale (GCS) score is a "10." Which client assessment is most important for the reporting nurse to include? 1. Belief that the current surroundings are a racetrack 2. GCS score was "11" one hour ago 3. Recent vital signs show blood pressure of 120/80 mm Hg and pulse of 82/min 4. Reported allergy to penicillin and vancomycin

2 The GCS quantifies the level of consciousness in a client with acute brain injury by measuring eye opening (alertness), verbal response (orientation), and motor response (eg, obeying a command, frontal lobe function). The maximum score on the GCS is 15 and the lowest is 3. If a client is trending for deterioration, this should always be noted in neurological assessments. A numerical decline of a single number in 1 hour is significant. A criticism of the GCS score is that it is not that precise. (Option 1) Orientation to place is part of the GCS score (under best verbal). The total score and the negative trend are more indicative of the client's condition than any individual GCS component. (Option 3) This client's vital signs are within normal limits and are not significant. It would be more important to communicate if there is absence of Cushing's triad (bradycardia, bradypnea/Cheyne-Stokes, and widening pulse pressure) or to give a brief summary of overall vital signs (eg, "normal"); exact readings are accessible and can be recited if the oncoming nurse needs to know them. However, reporting the negative neurological trend in the GCS score is the priority. (Option 4) Although it is important to be aware of allergies, the oncoming nurse can find that information on the chart if these medications are ordered. The main concern is blunt head trauma and not infection; therefore, it is unlikely that the oncoming nurse will be required to know this information and need to administer antibiotics. Educational objective: It is a priority to report a negative neurological trend as evidenced by GCS score in a client with blunt head trauma.

The nurse is caring for a client with a history of headaches. The client has talked to the nurse, smiled at guests, and maintained stable vital signs. The nurse notes the following changes in the client's status. Which assessment finding is critical to report to the health care provider (HCP)? 1. Blood pressure 136/88 mm Hg 2. Flat affect and drowsiness 3. Poor appetite 4. Respiratory rate 12/min

2 The level of consciousness is the most important, sensitive, and reliable indicator of the client's neurological status. Changes in the level of consciousness can represent increased intracranial pressure and reduced cerebral blood flow. Changes in vital signs usually do not appear until intracranial pressure has been elevated for some time, or they may be sudden in cases of head trauma. (Option 1) The blood pressure is slightly elevated but does not warrant immediate action or signify an emergency situation. (Option 3) A poor appetite is not an emergency finding or situation. (Option 4) The respiratory rate is slightly low, but if it is not irregular it is not an emergency as a single observation. This finding would warrant further assessment and continued monitoring, but it is not as significant as the change in level of consciousness. Educational objective: A change in level of consciousness for the neurological client should be reported to the HCP. The level of consciousness is the most sensitive and reliable indicator of the client's neurological status.

The nurse is assessing a client with advanced amyotrophic lateral sclerosis. Which of the following assessment findings does the nurse expect? Select all that apply. 1. Diarrhea 2. Difficulty breathing 3. Difficulty swallowing 4. Muscle weakness 5. Resting tremor

2, 3, 4 Amyotrophic lateral sclerosis (ALS, Lou Gehrig disease) is a debilitating neurodegenerative disease with no cure. ALS causes progressive degeneration of motor neurons in the brain and spinal cord. Physical symptoms include fatigue, progressive muscle weakness, twitching and muscle spasms, difficulty swallowing, difficulty speaking, and respiratory failure (Options 2, 3, and 4). Most clients survive only 3-5 years after the diagnosis as there is no cure. Treatment focuses on symptom management. Interventions include: - Respiratory support with noninvasive positive pressure (eg, bilevel positive airway pressure [BiPAP]) or invasive mechanical ventilation (eg, via tracheostomy) - Feeding tube for enteral nutrition - Medications to decrease symptoms (eg, spasms, uncontrolled secretions, dyspnea) - Mobility assistive devices (eg, walker, wheelchair) - Communication assistive devices (eg, alphabet boards, specialized computers) (Option 1) Constipation due to decreased mobility is more common in ALS. Diarrhea is not seen. (Option 5) Resting tremor is characteristic of parkinsonism. Educational objective: Amyotrophic lateral sclerosis causes motor neuron degeneration that leads to progressive muscle weakness, twitching and muscle spasms, difficulty swallowing, difficulty speaking, and respiratory failure. There is no cure. Treatment focuses on symptom management.

The nurse is caring for a client diagnosed with Broca aphasia due to a stroke. Which of the following deficits would the nurse correctly attribute to Broca aphasia? Select all that apply. 1. Client coughs and gasps when swallowing food and liquids 2. Client is easily frustrated while attempting to speak 3. Client is unable to understand speech and is completely nonverbal 4. Client misunderstands and inappropriately responds to verbal instruction 5. Client's speech is limited to short phrases that require effort

2, 5 Broca (expressive) aphasia is a nonfluent aphasia resulting from damage to the frontal lobe. Clients with Broca aphasia can comprehend speech but demonstrate speech difficulties. The speech pattern often consists of short, limited phrases that make sense but display great effort and frequent omission of smaller words (eg, "and," "is," "the") (Option 5). Clients with Broca aphasia are aware of their deficits and can become frustrated easily (Option 2). In comparison, clients with Wernicke (receptive) aphasia are unaware of their speech impairment. (Option 1) Trouble swallowing, often identified by coughing and gasping when eating and drinking, is dysphagia, which is not related to Broca aphasia. (Option 3) Clients with damage to multiple language areas of the brain may develop global aphasia, resulting in the inability to read, write, or understand speech. This is the most severe form of aphasia. (Option 4) Clients with damage to the temporal portion of the brain may develop Wernicke (fluent) aphasia (ie, the inability to comprehend the spoken and/or written word) and exhibit a long, but meaningless, speech pattern. Educational objective: Damage to the frontal lobe of the brain may cause Broca (expressive) aphasia. Clients with this condition demonstrate effortful and sensible speech characterized by short, limited sentences, with retained ability to comprehend speech. This impairment often causes clients with Broca aphasia to be frustrated when speaking.

The nurse is caring for a client with a history of tonic-clonic seizures. After a seizure lasting 25 seconds, the nurse notes that the client is confused for 20 minutes. The client does not know the current location, does not know the current season, and has a headache. The nurse documents the confusion and headache as which phase of the client's seizure activity? 1. Aural phase 2. Ictal phase 3. Postictal phase 4. Prodromal phase

3 A seizure is an uncontrolled electrical discharge of neurons in the brain that interrupts normal function. Seizure manifestations generally are classified into 4 phases: 1) The prodromal phase is the period with warning signs that precede the seizure (before the aural phase). 2) The aural phase is the period before the seizure when the client may experience visual or other sensory changes. Not all clients experience or can recognize a prodromal or aural phase before the seizure. 3) The ictal phase is the period of active seizure activity. 4) During the postictal phase, the client may experience confusion while recovering from the seizure. The client may also experience a headache. Postictal confusion can help identify clients by differentiating seizures from syncope. In syncope, there will be only a brief loss of consciousness without prolonged post-event confusion. Educational objective: Clients may experience confusion after a seizure during the postictal phase. The client should be observed for safety and abnormalities documented before and during this phase.

The nurse is caring for an assigned team of clients. Which client is the priority for the nurse at this time? 1. Client admitted with Guillain-Barré syndrome yesterday is paralyzed to the knees 2. Client admitted with multiple sclerosis exacerbation has scanning speech 3. Client with epilepsy puts on call light and reports having an aura 4. Client with fibromyalgia reports pain in the neck and shoulders

3 An aura is a sensory perception that occurs prior to a complex or generalized seizure. The client will most likely have a tonic-clonic seizure soon, and the nurse should attend to this client first to ensure safety measures (ie, seizure precautions) are in place. (Option 1) Guillain-Barré syndrome is an ascending symmetrical paralysis. It can move upward rapidly or relatively slowly (over days/weeks). Respiratory compromise is the worst complication. A client with paralysis at the level of the knee after 24 hours would not take priority over a client who will have a seizure in few minutes. (Option 2) Scanning speech is a dysarthria in which there are noticeable pauses between syllables and/or emphasis on unusual syllables. It is an expected finding with multiple sclerosis. (Option 4) Fibromyalgia involves neuroendocrine/neurotransmitter dysregulation. Clients experience widespread pain with point tenderness at multiple sites, including the neck and shoulders. This client is not a priority. Educational objective: An aura is a sensory warning that a complex or generalized seizure will occur. It is a priority over stable or expected findings such as point tenderness in fibromyalgia, low-level location of paralysis in Guillain-Barré syndrome, or scanning speech in multiple sclerosis.

The nurse receives the assigned clients for today on a neurology unit. The nurse should check on which client first? 1. Client with history of head injury whose Glasgow Coma Scale (GCS) changes from 13 to 14 2. Client with history of myasthenia gravis who had ptosis in the evening 3. Client with history of T2 spinal injury who has diaphoresis, pulse 54/min, and hypertension 4. Client with history of transverse myelitis with 2+ bilateral lower extremity muscle strength

3 Autonomic dysreflexia (autonomic hyperreflexia) is a massive, uncompensated cardiovascular reaction by the sympathetic nervous system (SNS) in a spinal injury at T6 or higher. Due to the injury, the parasympathetic nervous system cannot counteract the SNS stimulation below the injury. Classic triggers are distended bladder or rectum. Classic manifestations include severe hypertension, throbbing headache, marked diaphoresis above the level of injury, bradycardia, piloerection (goose bumps), and flushing. This is an emergency condition requiring immediate intervention. Management includes raising the head of the bed and then treating the cause. (Option 1) The Glasgow Coma Scale (GCS) is an objective scale used to monitor alertness/mental functioning in an acute head injury. The best score is 15, and the worst is 3. This client has an improving trend and is not a priority. (Option 2) Myasthenia gravis is an autoimmune disease manifesting mainly as muscle weakness and ptosis. The muscle weakness increases with activity, and by the end of the day, ptosis is present. These are expected findings for this condition, and so this client is not a priority. However, clients with myasthenic crisis can have respiratory failure, which, if it occurs, would be a priority. (Option 4) Transverse myelitis (spinal cord inflammation) usually results from a recent viral infection. Classic symptoms include paralysis, urinary retention, and bowel incontinence. Some clients recover, but many have permanent disability. Normal muscle strength is 5 on a scale of 0-5. Weakened muscle strength (2+ means only able to move laterally, not able to lift up against gravity) would be an expected finding. Educational objective: Autonomic dysreflexia in a client with a spinal cord injury is a priority and requires emergency intervention. Classic triggers are distended bladder or rectum. Management includes raising the head of the bed and then treating the cause (eg, Foley catheter kinks).

A client with a history of headaches is scheduled for a lumbar puncture to assess the cerebrospinal fluid pressure. The nurse is preparing the client for the procedure. Which statement by the client indicates a need for further teaching by the nurse? 1. "I may feel a sharp pain that shoots to my leg, but it should pass soon." 2. "I will go to the bathroom and try to urinate before the procedure." 3. "I will need to lie on my stomach during the procedure." 4. "The physician will insert a needle between the bones in my lower spine."

3 Cerebrospinal fluid (CSF) is assessed for color, contents, and pressure. Normal CSF is clear and colorless, and contains a little protein, a little glucose, minimal white blood cells, no red blood cells, and no microorganisms. Normal CSF pressure is 60-150 mm H2O. Abnormal CSF pressure or contents can help diagnose the cause of headaches in complicated cases. CSF is collected via lumbar puncture or ventriculostomy. Prior to a lumbar puncture, clients are instructed as follows: - Empty the bladder before the procedure (Option 2) - The procedure can be performed in the lateral recumbent position or sitting upright. These positions help widen the space between the vertebrae and allow easier insertion of the needle (Option 3). - A sterile needle will be inserted between the L3/4 or L4/5 interspace (Option 4) - Pain may be felt radiating down the leg, but it should be temporary (Option 1) After the procedure, instruct the client as follows: - Lie flat with no pillow for at least 4 hours to reduce the chance of spinal fluid leak and resultant headache - Increase fluid intake for at least 24 hours to prevent dehydration Educational objective: Lumbar puncture can be performed with clients in the sitting position or positioned on the left side with the knees drawn up (fetal position).

The health care provider prescribes a multivitamin regimen that includes thiamine for a client with a history of chronic alcohol abuse. The nurse is aware that thiamine is given to this client population for which purpose? 1. To lower the blood alcohol level 2. To prevent gross tremors 3. To prevent Wernicke encephalopathy 4. To treat seizures related to acute alcohol withdrawal

3 Clients with chronic alcohol abuse suffer from poor nutrition related to improper diet and altered nutrient absorption. Poor thiamine intake and/or absorption can lead to Wernicke encephalopathy, a serious complication that manifests as altered mental status, oculomotor dysfunction, and ataxia. Clients are prescribed thiamine to prevent this condition. (Option 1) Thiamine is not used to reduce the blood alcohol level; there is no antidote for elevated blood alcohol levels. Supportive measures are used until levels fall and the client is no longer experiencing acute withdrawal symptoms. (Option 2) Gross tremors related to acute alcohol withdrawal are treated with benzodiazepines (eg, lorazepam, diazepam). (Option 4) Seizures related to acute alcohol withdrawal are treated with benzodiazepines (eg, lorazepam, diazepam, chlordiazepoxide). Thiamine is used to prevent encephalopathy. Encephalopathy may lead to seizures, but thiamine is not used to treat seizures. Educational objective: A major complication of chronic alcohol abuse is encephalopathy related to poor thiamine absorption. It is critical that these clients receive thiamine replacement. Wernicke encephalopathy can lead to more significant and progressive complications, including death.

A nurse is evaluating an acutely ill client with suspected meningitis. The nurse should take what action first? 1. Check for Kernig's and Brudzinski's signs 2. Establish IV access 3. Place the client on droplet precautions 4. Prepare the client for lumbar puncture

3 The client with suspected bacterial meningitis should be placed on droplet precaution isolation until the causative agent has been identified and appropriate treatment is initiated. Meningococcal meningitis and Haemophilus influenzae type B meningitis are highly transmissible to others, and the client must remain on droplet isolation until these can be ruled out. Precautions can usually be discontinued 24 hours after beginning antibiotic therapy. Viral meningitis and other types of bacterial meningitis (ie, other than meningococcal meningitis) usually do not require droplet precautions. (Option 1) Although assessment is a priority and meningeal signs should be checked, the nurse can only safely perform these assessments once droplet precautions are in place. (Options 2 and 4) A peripheral IV catheter should be inserted to provide fluids. Subsequently, preparation for lumbar puncture is needed. However, placing the client on isolation is a priority to protect the nurse and other clients and care providers. Educational objective: The client with suspected bacterial meningitis should be placed on droplet precaution isolation until the causative agent has been identified and appropriate treatment is initiated.

A client is admitted to the hospital for severe headaches. The client has a history of increased intracranial pressure (ICP), which has required lumbar punctures to relieve the pressure by draining cerebrospinal fluid. The client suddenly vomits and states, "That's weird, I didn't even feel nauseated." Which action by the nurse is the most appropriate? 1. Document the amount of emesis 2. Lower the head of the bed 3. Notify the health care provider (HCP) 4. Offer anti-nausea medication

3 Unexpected and projectile vomiting without nausea can be a sign of increased ICP, especially in the client with a history of increased ICP. The unexpected vomiting is related to pressure changes in the cranium. The vomiting can be associated with headache and gets worse with lowered head position. The most appropriate action is to obtain a full set of vital signs and contact the HCP immediately. (Option 1) Documentation is important, but it is not the priority action. (Option 2) The head of the bed should be raised, not lowered, for clients with suspected increased ICP. Raising the head of the bed to 30 degrees helps to drain the cerebrospinal fluid via the valve system without lowering the cerebral blood pressure. (Option 4) The vomiting is caused not by nausea but by pressure changes in the cranium. Anti-nausea medications are often not effective. Decreasing intracranial pressure will help the vomiting. Educational objective: Notify the HCP of signs/symptoms of increased ICP, including unexpected vomiting. The vomiting is often projectile, associated with headache, and gets worse with lowering the head position.

The nurse is planning care for a client with suspected stroke who has just arrived at the emergency department with slurred speech, facial drooping, and right arm weakness that began 1 hour ago. Which of the following interventions should the nurse anticipate including in the initial plan of care? Select all that apply. 1. Arrange for a speech pathologist consult 2. Discuss community resources with family 3. Obtain a STAT CT scan of the head 4. Perform a baseline neurologic assessment 5. Prepare to initiate alteplase within the next 3 hours

3, 4, 5 Strokes may be either ischemic or hemorrhagic. Ischemic stroke occurs when circulation to parts of the brain is interrupted by occlusion of cerebral blood vessels by a thrombosis or embolus. Hemorrhagic stroke occurs when a cerebral blood vessel ruptures and bleeds into the cranial vault. Both types of stroke result in brain tissue death without prompt treatment. - A client with stroke symptoms must have an immediate CT scan or MRI of the head to determine the type and location of the stroke (Option 3). Determining exactly when symptoms began is essential for diagnosis and planning treatment. - Thrombolytic therapy (eg, alteplase, tissue plasminogen activator [tPA]) is used to dissolve blood clots and restore perfusion to brain tissue in clients with an ischemic stroke unless contraindicated (eg, active bleeding, uncontrolled hypertension, aneurysm). It must be administered within 4.5 hours from onset of symptoms (Option 5). - A baseline neurologic assessment is essential for tracking ongoing neurologic symptoms that indicate improvement or complications which guide later treatments (Option 4). (Options 1 and 2) Consultation with a speech pathologist and providing the family with information about community resources are important later but not during the initial (acute) phase of stroke management. Educational objective: The initial plan of care for a client with an acute stroke should include performing baseline neurologic assessment to begin monitoring neurologic status trend, obtaining an immediate CT scan of the head to determine stroke type, and anticipating administration of thrombolytics (if indicated) within 4.5 hours of symptom onset.

The nurse is providing discharge education for a postoperative client who had a partial laryngectomy for laryngeal cancer. The client is concerned because the health care provider said there was damage to the ninth cranial nerve. Which statement made by the nurse is most appropriate? 1. "I will ask the health care provider to explain the consequences of your procedure." 2. "This is a common complication that will require you to have a hearing test every year." 3. "This is a common complication; your health care provider will order a consult for the speech pathologist." 4. "This is the reason you are using a special swallowing technique when you eat and drink."

4 Cranial nerve IX (glossopharyngeal) is involved in the gag reflex, ability to swallow, phonation, and taste. Postoperative partial laryngectomy clients will need to undergo evaluation by a speech pathologist to evaluate their ability to swallow safely to prevent aspiration. Clients are taught the supraglottic swallow, a technique that allows them to have voluntary control over closing the vocal cords to protect themselves from aspiration. Clients are instructed to: - Inhale deeply - Hold breath tightly to close the vocal cords - Place food in mouth and swallow while continuing to hold breath - Cough to dispel remaining food from vocal cords - Swallow a second time before breathing (Option 1) This would be considered "passing the buck." The nurse should try to address the client's concerns before calling the health care provider. (Option 2) Cranial nerve VIII (vestibulocochlear) affects hearing and equilibrium, not swallowing. (Option 3) The speech pathologist conducts a swallowing assessment early on to evaluate a client's ability to swallow safely. This consult is not done at discharge. Educational objective: Clients who undergo a partial laryngectomy are at increased risk for aspiration. As a result, they are taught a swallowing technique (supraglottic swallow) to decrease this risk.

The emergency department nurse is triaging clients. Which neurologic presentation is most concerning for a serious etiology and should be given priority for definitive treatment? 1. History of Bell's palsy with unilateral facial droop and drooling 2. History of multiple sclerosis and reporting recent blurred vision 3. Reports unilateral facial pain when consuming hot foods 4. Temple region hit by ball, loss of consciousness, but Glasgow Coma Scale score is now 14

4 Epidural hematoma is an accumulation of blood between the skull bone and dura mater. The majority of epidural hematomas are associated with fracture of the temporal bone and subsequent rupture or tear of the middle meningeal artery. The bleed is arterial in origin, and so hematoma develops quickly. The clinical presentation of epidural hematoma is characteristic. The client may lose consciousness at the time of impact. The client then regains consciousness quickly and feels well for some time after the injury. This transient period of well-being is called a lucid interval. It is followed by a quick decline in mental function that can progress into coma and death. (Option 1) Bell's palsy (peripheral facial paralysis) is an inflammation of the facial nerve (CN VII) in the absence of other disease etiologies, such as stroke. There is flaccidity of the affected side with drooling. This differs from the concerning drooling with epiglottis in which the client's throat is too sore and/or swollen to swallow saliva. Treatment includes steroids, measures to relieve symptoms, and protection of the eye (which may not close tightly), but the condition is not emergent. (Option 2) Multiple sclerosis is a chronic, relapsing, and remitting degenerative disorder involving the brain, optic nerve, and spinal cord. Optic neuritis is a common presentation but is not life-threatening. (Option 3) Trigeminal neuralgia (tic douloureux) presents with paroxysms of unilateral excruciating facial pain along the distribution of the trigeminal nerve (CN V) that are often triggered by touch, talking, or hot/cold air or intake. Carbamazepine (Tegretol) is the drug of choice; the condition is not life-threatening. Educational objective: The classic presentation of intracranial epidural bleed is loss of consciousness to a period of lucidity and then gradual loss of consciousness. The bleed is arterial in origin, and so hematoma develops quickly. Emergent diagnosis and treatment are needed to prevent brain stem herniation.

A client with multiple sclerosis is voicing concerns to the nurse about incoordination when walking. Which of the following instructions by the nurse would be most appropriate at this time? 1. "Avoid excess stretching of your lower extremities." 2. "Build strength by increasing the duration of daily exercise." 3. "Let me speak with your health care provider about getting a wheelchair." 4. "You should keep your feet apart and use a cane when walking."

4 Multiple sclerosis (MS) is a progressive, demyelinating disease of the central nervous system that interrupts nerve impulses, causing a variety of symptoms. Symptoms may vary, but muscle weakness, spasticity, incoordination, loss of balance, and fatigue are usually present, causing impaired mobility and risk for fall and injury. Walking with the feet apart increases the support base, improving steadiness and gait. Assistive devices, such as a cane or walker, are usually required as demyelination of the nerve fibers progresses. (Option 1) Range-of-motion, strengthening, and stretching exercises help limit spasticity and contractures in clients with MS. (Option 2) Fatigue is a common symptom with MS. Rather than increasing the duration, clients should balance exercise with rest. Clients should also exercise when the weather is cool and stay hydrated; dehydration and extremes in temperature cause symptom exacerbation. (Option 3) Wheelchairs are advised only if exercise and gait training are not successful as clients should maintain mobility and independence as long as possible. Educational objective: Clients with multiple sclerosis experience fatigue, incoordination, balance impairment, muscle weakness, and muscle spasticity from demyelination of nerve fibers. Gait training (eg, walking with the feet apart) and assistive devices can help prevent falls and injury and preserve independence as long as possible.

A nurse cares for a client with impairment of cranial nerve VIII. What instructions will the nurse provide the unlicensed assistive personnel prior to delegating interventions related to the client's activities of daily living? 1. "Be aware of the client's shoulder weakness and provide support as needed." 2. "Ensure that the client sits upright and tucks the chin when swallowing food." 3. "Explain all procedures in step-by-step detail before performing them." 4. "Make sure the items needed by the client are within reach."

4 The client has an impairment of cranial nerve (CN) VIII, the vestibulocochlear (or auditory) nerve. Symptoms of impairment may include loss of hearing, dizziness, vertigo, and motion sickness, which place the client at a high risk for falls. Therefore, when instructing the unlicensed assistive personnel (UAP) about helping the client with activities of daily living, the nurse emphasizes the need to keep items at the bedside within the client's reach (Option 4). (Option 1) Weakness of the shoulder muscle occurs with impairment of CN XI, the spinal accessory nerve. Impairment of CN VIII does not affect shoulder strength. (Option 2) Dysphagia may occur with impairment of CN IX (glossopharyngeal) and CN X (vagus), not CN VIII. Instructing the client to tuck the chin while eating is a technique for those who have difficulty swallowing. (Option 3) Impairment of visual acuity occurs with disorders affecting CN II (optic). Because impairment of CN VIII does not affect visual acuity, providing a detailed, step-by-step explanation of procedures may be helpful but is not the most appropriate instruction to give the UAP. Educational objective: Impairment of cranial nerve (CN) VIII, the vestibulocochlear or auditory nerve, may cause dizziness, vertigo, loss of hearing, and motion sickness. To assist the client with impairment of CN VIII, needed items should be placed nearby to decrease the risk of the client getting out of bed and falling.

The nurse is assessing the cranial nerves and begins testing the facial nerve (cranial nerve VII). Which direction should the nurse give the client to test this cranial nerve? 1. "Close your eyes and identify this smell." 2. "Follow my finger with your eyes without moving your head." 3. "Look straight ahead and let me know when you can see my finger." 4. "Raise your eyebrows, smile, and frown."

4 The facial nerve, cranial nerve VII, is tested by assessing exaggerated facial movements. The client is directed to raise the eyebrows, furrow the eyebrows, draw up the cheeks in a large smile, pull the cheeks down in a frown, and open the lips to show the teeth. Any asymmetrical movements are documented, and if unexpected, the health care provider is notified. (Option 1) Cranial nerve I is the olfactory sensory nerve. This nerve is tested by having the client identify a readily recognized odor. (Option 2) Cranial nerve III is a motor nerve of the eye, which is tested by having the client track an object, such as a finger, through the fields of vision. (Option 3) Cranial nerve II is the optic nerve and is a sensory nerve. It is assessed by testing the fields of vision for the client's ability to see objects in the field. Educational objective: The facial nerve, cranial nerve VII, is tested by assessing exaggerated facial movements. The client is directed to raise the eyebrows, furrow the eyebrows, draw up the cheeks in a large smile, pull the cheeks down in a frown, and open the lips to show the teeth.

The nurse in the outpatient clinic is speaking with a client diagnosed with cerebral arteriovenous malformation. Which statement would be a priority for the nurse to report to the health care provider? 1. "I got short of breath this morning when I worked out." 2. "I have cut down on smoking to 1/2 pack per day." 3. "I haven't been feeling well, so I have been sleeping a lot." 4. "I took an acetaminophen in the waiting room for this bad headache."

4 An arteriovenous malformation (AVM) is a tangle of veins and arteries that is believed to form during embryonic development. The tangled vessels do not have a capillary bed, causing them to become weak and dilated. AVMs are usually found in the brain and can cause seizures, headaches, and neurologic deficits. Treatment depends on the location of the AVM, but blood pressure control is crucial. Clients with AVMs are at high risk for having an intracranial bleed as the veins can easily rupture because they lack a muscular layer around their lumen. Any neurologic changes, sudden severe headache, nausea, and vomiting should be evaluated immediately as these are usually the first symptoms of a hemorrhage (Option 4). (Option 1) The report of dyspnea may prompt further evaluation depending on the type of exercise performed, but it is not the priority. Clients with AVMs should be discouraged from engaging in heavy exercise as it increases blood pressure. (Option 2) Clients with AVMs should avoid smoking to prevent hypertension. This client needs education on smoking cessation, but it is not the priority. (Option 3) Reports of not feeling well and sleeping a lot may be related to the headache and possible hemorrhage, but this alone would not prompt a call to the health care provider. Educational objective: An arteriovenous malformation is a congenital deformity of tangled blood vessels often occurring in the brain. These vessels may weaken and rupture, causing an intracranial hemorrhage. Any neurologic changes and severe headache need to be addressed immediately as these may indicate hemorrhage.

The clinic nurse is assessing a previously healthy 60-year-old client when the client says, "My hand has been shaking when I try to cut food. I did some research online. Could I have Parkinson's disease?" Which response from the nurse is the most helpful? 1. "It can't be Parkinson's disease because you aren't old enough." 2. "Make sure you tell the physician about your concerns." 3. "Parkinson's disease does not cause that kind of hand shaking." 4. "Tell me more about your symptoms. When did they start?"

4 Parkinson's disease (PD) is a chronic, progressive neurodegenerative disorder that involves degeneration of the dopamine-producing neurons. Damage to dopamine neurons makes it difficult to control muscles through smooth movement. PD is characterized by a delay in initiation of movement (bradykinesia), increased muscle tone (rigidity), resting tremor, and shuffling gait. The most helpful response by the nurse is the one that acknowledges the concern of the client and also asks for more information. The nurse should assess for additional information and perform a more focused physical assessment given this new information (Option 4). (Option 1) It is incorrect to say that the client is too young to have PD although it is usually seen after age 60; about 15% of PD cases are diagnosed before age 50. (Option 2) Although the nurse should encourage the client and family to discuss concerns with the health care provider, this is not the most helpful response. (Option 3) Although the typical parkinsonian tremor occurs at rest and not during purposeful movement, it is not helpful to dismiss a concern without probing for more information. Educational objective: Therapeutic communication includes acknowledging concerns and probing for additional information as part of an assessment.

The nurse is caring for a client who has homonymous hemianopsia following an acute stroke. Which nursing diagnosis is the most appropriate for this client? 1. Risk for ineffective airway maintenance 2. Risk for knowledge deficit 3. Risk for poor fluid intake 4. Risk for self-neglect

4 Cerebral vascular accidents (strokes) can cause visual and perceptual deficits depending on which part of the brain is affected. Clients with changes in visual field or perception of their body in space can be at risk for safety-related injuries. Homonymous hemianopsia is a loss in half of the visual field on the same side. For example, the client may lose the left side of the visual field in both eyes. A client unable to see the left side of the body is at a higher risk for neglecting that side or being unable to eat food placed on the left side of a plate. These clients are at higher risk for injury because they are unable to incorporate full visual field input. They are taught to turn the head and scan to the side with the visual field deficit to reduce the risk for injury and self-neglect. (Option 1) Clients with a reduced visual field are not at a higher risk for ineffective airway maintenance. (Option 2) Clients with a reduced visual field are at risk for knowledge deficit and should be taught to turn the head and scan the area with the deficit. However, this risk is not more important than client safety and the risk of self-neglect. (Option 3) Clients with a reduced visual field are not specifically at higher risk for reduced fluid intake. Food and fluids should be kept within the client's field of vision to encourage intake as appropriate. Educational objective: Homonymous hemianopsia is the loss of one half of the field of vision on the same side in both eyes. Clients are taught to turn their heads and scan the area that has a visual deficit to reduce the risk of injury and self-neglect. Food and fluids should be kept within the client's field of vision to encourage intake as appropriate.

A client is being admitted for a potential cerebellar pathology. Which tasks should the nurse ask the client to perform to assess if cerebellar function is within the defined limits? Select all that apply. 1. Identify the number "8" traced on the palm 2. Shrug the shoulders against resistance 3. Swallow water 4. Touch each finger of one hand to the hand's thumb 5. Walk heel-to-toe

4, 5 The cerebellum is involved in 2 major functions: coordination of voluntary movements and maintenance of balance and posture. >Maintenance of balance is assessed with gait testing and includes watching the client's normal gait first and then the gait on heel-to-toe (tandem), on toes, and on heels (Option 5). >Coordination testing involves the following: - Finger tapping - ability to touch each finger of one hand to the hand's thumb (Option 4). - Rapid alternating movements - rapid supination and pronation - Finger-to-nose testing - clients touch the clinician's finger and then their own nose as the clinician's finger varies in location - Heel-to-shin testing - client runs each heel down each shin while in a supine position (Option 1) This is a test of sensory function, specifically fine touch (graphesthesia). Other tests for this include identifying an object in the hand (stereognosis) and two-point discrimination. (Option 2) Shrugging the shoulders against resistance (as well as turning the head against resistance) is a test for cranial nerve (CN) XI (spinal accessory). (Option 3) In a client who has an intact gag reflex, the ability to swallow water helps to assess CN IX (glossopharyngeal) and CN X (vagus). The nurse can also observe for a symmetrical rise of the soft palate and uvula by asking the client to say "ah." Educational objective: The cerebellum is involved in coordination of voluntary movements and maintenance of balance and posture. Balance is assessed with heel-to-toe gait testing. Coordination is assessed with finger tapping, rapid alternating movements, finger-to-nose testing, and heel-to-shin testing.

The nurse moves a finger in a horizontal and vertical motion in front of the client's face while directing the client to follow the finger with the eyes. Which cranial nerves is the nurse assessing? Select all that apply. 1. II 2. III 3. IV 4. V 5. VI

Cranial nerves III, IV, VI The oculomotor (cranial nerve III), trochlear (cranial nerve IV), and abducens (cranial nerve VI) are motor nerves of the eye that are tested by having the client track an object, such as a finger, through the fields of vision. The oculomotor nerve is also tested by checking for pupillary constriction and accommodation (constriction with near vision). Deficits in cranial nerves III, IV, and VI can include disconjugate gaze (eyes do not move together), nystagmus (fine, rapid jerking eye movements), or ptosis (drooping of the eyelid). (Option 1) Cranial nerve II is the optic nerve and a sensory nerve. This nerve is assessed by testing the fields of vision for the client's ability to see objects in the field. In contrast to cranial nerves III, IV, and VI, the client does not track the object in the fields of vision, but instead keeps the eyes fixed and uses the peripheral vision to recognize objects or deficits in the field of vision. (Option 4) Cranial nerve V is the trigeminal nerve. The sensory portion of this nerve is assessed by testing sensation at the ophthalmic (forehead), maxillary (cheekbone), and mandibular (jaw line) branches by light touch. Corneal sensation is also a portion of the trigeminal nerve, but this is typically not tested by the nurse. Educational objective: Motor function of the eyes is tested by having the client track an object through the fields of vision and documenting any abnormalities in eye movement that represent cranial nerves III, IV, and VI. The health care provider is then notified of the findings.


Conjuntos de estudio relacionados

EDU 300 - CHAPTER 14 Learners with Physical Disabilities and Other Health Impairments

View Set

Ch: 21 Care of the maternal newborn

View Set

Anatomy Blood Pretest 4 Answers Reonisto

View Set

Chapter 32 disorders of endocrine control

View Set

National Electric Code NEC - Article 310

View Set

Nursing 503 Exam 3 Flashcards from ATI

View Set

Final Exam 2019 Business Management

View Set